- PowerScore Staff
- Posts: 5972
- Joined: Mar 25, 2011
- Thu Jul 14, 2016 3:34 pm
#27189
Complete Question Explanation
(The complete setup for this game can be found here: lsat/viewtopic.php?t=11429)
The correct answer choice is (B)
Irrespective of the Local conditions, answer choice (C) can be eliminated by applying the Not Laws.
Since neither F nor N is scheduled, it follows that H, L, O, P, S, and W must be scheduled. And since F is not scheduled, P cannot be scheduled for day 1 or day 2 (the contrapositive of the final rule). Consequently P must be scheduled for day 3 and thus answer choice (D) is incorrect. Since O is scheduled for day 2 and P is scheduled for day 3, the third rule can be applied, and it follows that since L cannot be scheduled with O or P, then L must be scheduled for day 1. Thus, answer choice (B) is correct.
(The complete setup for this game can be found here: lsat/viewtopic.php?t=11429)
The correct answer choice is (B)
Irrespective of the Local conditions, answer choice (C) can be eliminated by applying the Not Laws.
Since neither F nor N is scheduled, it follows that H, L, O, P, S, and W must be scheduled. And since F is not scheduled, P cannot be scheduled for day 1 or day 2 (the contrapositive of the final rule). Consequently P must be scheduled for day 3 and thus answer choice (D) is incorrect. Since O is scheduled for day 2 and P is scheduled for day 3, the third rule can be applied, and it follows that since L cannot be scheduled with O or P, then L must be scheduled for day 1. Thus, answer choice (B) is correct.
Dave Killoran
PowerScore Test Preparation
Follow me on X/Twitter at http://twitter.com/DaveKilloran
My LSAT Articles: http://blog.powerscore.com/lsat/author/dave-killoran
PowerScore Podcast: http://www.powerscore.com/lsat/podcast/
PowerScore Test Preparation
Follow me on X/Twitter at http://twitter.com/DaveKilloran
My LSAT Articles: http://blog.powerscore.com/lsat/author/dave-killoran
PowerScore Podcast: http://www.powerscore.com/lsat/podcast/